Re: [Vo]:Interstellar travel

2020-10-12 Thread Jed Rothwell
Okay, to summarize this discussion:

A radar would not be able to detect particles such as grains of sand or
small rocks. Therefore I suppose you need a powerful laser in the front of
the spacecraft to heat them and break them into molecules or plasma. It
would have to be turned on at all times because you cannot detect these
particles.

Even atoms and molecules would be large enough to cause damage. So you need
to push them out of the way. I suppose the only way to do this with known
science is to ionize them and then use a magnetic field to push them out of
the way. I do not know if you can ionize particles from a long distance
away. It would have to be very long!

Something like a science fiction "force field" would probably be the best
solution. I have no idea whether such a thing is possible.


Re: [Vo]:Acoustic demonstration of beats

2020-10-12 Thread Jones Beene
 

The appearance of higher amplitude sound waves could seem, at first, like a 
path to net gain.
Dicke "superradiance is involved as well as Fermi-Pasta-Ulam

https://en.wikipedia.org/wiki/Fermi%E2%80%93Pasta%E2%80%93Ulam%E2%80%93Tsingou_problem

H LV wrote:  
 
Acoustic demonstration of beats
https://www.youtube.com/watch?v=IYeV2Wq82fw

This is not mentioned in the video but beats also arise and persist in a driven 
oscillator when no damping force is present. This happens because the driver`s 
frequency does not match the natural frequency of the oscillator. Beats will 
also initially appear in a driven oscillator when a damping force is present 
but they will fade away quickly.
Harry


  

Re: [Vo]:Acoustic demonstration of beats

2020-10-12 Thread H LV
On Mon, Oct 12, 2020 at 11:12 AM Jones Beene  wrote:

>
>
> The appearance of higher amplitude sound waves could seem, at first, like
> a path to net gain.
>
> Dicke "superradiance is involved as well as Fermi-Pasta-Ulam
>
>
> https://en.wikipedia.org/wiki/Fermi%E2%80%93Pasta%E2%80%93Ulam%E2%80%93Tsingou_problem
>
>
Could this mean that under the right conditions a body could
unexpectedly radiate more of its energy in the infrared region?

Harry




>
> H LV wrote:
>
>
> Acoustic demonstration of beats
> https://www.youtube.com/watch?v=IYeV2Wq82fw
>
> This is not mentioned in the video but beats also arise and persist in a
> driven oscillator when no damping force is present. This happens because
> the driver`s frequency does not match the natural frequency of the
> oscillator. Beats will also initially appear in a driven oscillator when a
> damping force is present but they will fade away quickly.
>
> Harry
>
>
>
>


RE: [Vo]:Acoustic demonstration of beats

2020-10-12 Thread JonesBeene

Yes. For instance, if your expectation is based on emission from  a stationary 
emitter – then  “ rotational superradiance” can alter and  concentrate 
radiation from around the equator of the rapidly spinning emitter while the 
polar emission will be subradiant. No gain – simply a shift.



The appearance of higher amplitude sound waves could seem, at first, like a 
path to net gain.

Dicke "superradiance is involved as well as Fermi-Pasta-Ulam

https://en.wikipedia.org/wiki/Fermi%E2%80%93Pasta%E2%80%93Ulam%E2%80%93Tsingou_problem


Could this mean that under the right conditions a body could unexpectedly 
radiate more of its energy in the infrared region?

Harry 


H LV wrote: 


Acoustic demonstration of beats
https://www.youtube.com/watch?v=IYeV2Wq82fw

This is not mentioned in the video but beats also arise and persist in a driven 
oscillator when no damping force is present. This happens because the driver`s 
frequency does not match the natural frequency of the oscillator. Beats will 
also initially appear in a driven oscillator when a damping force is present 
but they will fade away quickly.

Harry






Re: [Vo]:Acoustic demonstration of beats

2020-10-12 Thread H LV
I wonder if quanta are made from beat frequencies:

hf  = h|f1- f2|

where  |f1- f2|  is the absolute value of the difference between the
frequencies of two interfering *classical* vibrations.

Perhaps ordinarily the existence of the other frequencies produce no
observable effects but under special circumstances
they do.

What I am proposing is related to the phenomena of quantum beats but
quantum beats involve the interference of two quanta or photons rather than
trying to explain why quanta exist.

This article gives a history of the subject and interestingly the existence
of quantum beats has often been denied because semi-classical quantum
theory says they are impossible but quantum electrodynamics does predict
them.
https://bayes.wustl.edu/etj/articles/quantum.beats.pdf

BTW the beat concept also played a role in thought of Louis de Broglie.

Harry

On Mon, Oct 12, 2020 at 3:16 PM JonesBeene  wrote:

>
>
> Yes. For instance, if your expectation is based on emission from  a
> stationary emitter – then  “ rotational superradiance” can alter and
>  concentrate radiation from around the equator of the rapidly spinning
> emitter while the polar emission will be subradiant. No gain – simply a
> shift.
>
>
>
>
>
>
>
> The appearance of higher amplitude sound waves could seem, at first, like
> a path to net gain.
>
>
>
> Dicke "superradiance is involved as well as Fermi-Pasta-Ulam
>
>
>
>
> https://en.wikipedia.org/wiki/Fermi%E2%80%93Pasta%E2%80%93Ulam%E2%80%93Tsingou_problem
>
>
>
>
>
> Could this mean that under the right conditions a body could
> unexpectedly radiate more of its energy in the infrared region?
>
>
>
> Harry
>
>
>
>
>
> H LV wrote:
>
>
>
>
>
> Acoustic demonstration of beats
>
> https://www.youtube.com/watch?v=IYeV2Wq82fw
>
>
>
> This is not mentioned in the video but beats also arise and persist in a
> driven oscillator when no damping force is present. This happens because
> the driver`s frequency does not match the natural frequency of the
> oscillator. Beats will also initially appear in a driven oscillator when a
> damping force is present but they will fade away quickly.
>
>
>
> Harry
>
>
>
>
>
>
>
>
>


Re: [Vo]:Acoustic demonstration of beats

2020-10-12 Thread H LV
Beat frequencies of two lasers irradiating a surface appear in
_Stimulation of Optical Phonons in Deuterated Palladium_ by Dennis Letts
and Peter Hagelstein
https://www.lenr-canr.org/acrobat/LettsDstimulatio.pdf

Harry

On Mon, Oct 12, 2020 at 11:20 PM H LV  wrote:

> I wonder if quanta are made from beat frequencies:
>
> hf  = h|f1- f2|
>
> where  |f1- f2|  is the absolute value of the difference between the
> frequencies of two interfering *classical* vibrations.
>
> Perhaps ordinarily the existence of the other frequencies produce no
> observable effects but under special circumstances
> they do.
>
> What I am proposing is related to the phenomena of quantum beats but
> quantum beats involve the interference of two quanta or photons rather than
> trying to explain why quanta exist.
>
> This article gives a history of the subject and interestingly the
> existence of quantum beats has often been denied because semi-classical
> quantum theory says they are impossible but quantum electrodynamics does
> predict them.
> https://bayes.wustl.edu/etj/articles/quantum.beats.pdf
>
> BTW the beat concept also played a role in thought of Louis de Broglie.
>
> Harry
>
> On Mon, Oct 12, 2020 at 3:16 PM JonesBeene  wrote:
>
>>
>>
>> Yes. For instance, if your expectation is based on emission from  a
>> stationary emitter – then  “ rotational superradiance” can alter and
>>  concentrate radiation from around the equator of the rapidly spinning
>> emitter while the polar emission will be subradiant. No gain – simply a
>> shift.
>>
>>
>>
>>
>>
>>
>>
>> The appearance of higher amplitude sound waves could seem, at first, like
>> a path to net gain.
>>
>>
>>
>> Dicke "superradiance is involved as well as Fermi-Pasta-Ulam
>>
>>
>>
>>
>> https://en.wikipedia.org/wiki/Fermi%E2%80%93Pasta%E2%80%93Ulam%E2%80%93Tsingou_problem
>>
>>
>>
>>
>>
>> Could this mean that under the right conditions a body could
>> unexpectedly radiate more of its energy in the infrared region?
>>
>>
>>
>> Harry
>>
>>
>>
>>
>>
>> H LV wrote:
>>
>>
>>
>>
>>
>> Acoustic demonstration of beats
>>
>> https://www.youtube.com/watch?v=IYeV2Wq82fw
>>
>>
>>
>> This is not mentioned in the video but beats also arise and persist in a
>> driven oscillator when no damping force is present. This happens because
>> the driver`s frequency does not match the natural frequency of the
>> oscillator. Beats will also initially appear in a driven oscillator when a
>> damping force is present but they will fade away quickly.
>>
>>
>>
>> Harry
>>
>>
>>
>>
>>
>>
>>
>>
>>
>